Problem understanding the Cantor Theorem's proof












0












$begingroup$


I can intuitevely picture the power set of A to be of a greater cardinality than A, as it permits multiple combinations of elements of A. However, I couldn't understand the usual proof that comes with the theorem. I tried looking for other proofs but couldn't find any. The proof assumes that in any bijective function f we choose, there's going to be elements of A that map into subsets they're not member of in P(A). And then we would use these as a contradiction.



My problem is the following: what if we choose a function f such that elements of A only map into subsets they're member of? Is this possible? Am I missing something? Wouldn't the proof fail with this kind of, but my understanding is that this f shouldn't exist, then why?



Thank you!










share|cite|improve this question









$endgroup$












  • $begingroup$
    I've never seen a proof like the one you describe. (Where did you find it?) The usual proof shows that, given any function $f:Ato P(A)$, we can find an element of $P(A)$ which is not in the range of $f$.
    $endgroup$
    – bof
    Jan 21 at 1:03










  • $begingroup$
    To do that, don't we usually define $B = { x in A | x notin f(x) } $ ?
    $endgroup$
    – bluemuse
    Jan 21 at 1:09










  • $begingroup$
    Indeed, so $B$ is an element of $P(A)$ which is not in the range of $f$. In your question you alleged that we find some element of $A$ with some funny property.
    $endgroup$
    – bof
    Jan 21 at 1:16












  • $begingroup$
    Are you confusing elements with subsets? $B$ is a subset of $A$, which makes it an element of $P(A)$.
    $endgroup$
    – bof
    Jan 21 at 1:21










  • $begingroup$
    But what if we choose an f such that B is empty? I mean an f such that elements of A always map into subsets containing them so the property of B ( $ x notin f(x) $ ) is not possible? Yes I might be confusing a lot of stuff here because everyone seems to understand this besides me
    $endgroup$
    – bluemuse
    Jan 21 at 1:24
















0












$begingroup$


I can intuitevely picture the power set of A to be of a greater cardinality than A, as it permits multiple combinations of elements of A. However, I couldn't understand the usual proof that comes with the theorem. I tried looking for other proofs but couldn't find any. The proof assumes that in any bijective function f we choose, there's going to be elements of A that map into subsets they're not member of in P(A). And then we would use these as a contradiction.



My problem is the following: what if we choose a function f such that elements of A only map into subsets they're member of? Is this possible? Am I missing something? Wouldn't the proof fail with this kind of, but my understanding is that this f shouldn't exist, then why?



Thank you!










share|cite|improve this question









$endgroup$












  • $begingroup$
    I've never seen a proof like the one you describe. (Where did you find it?) The usual proof shows that, given any function $f:Ato P(A)$, we can find an element of $P(A)$ which is not in the range of $f$.
    $endgroup$
    – bof
    Jan 21 at 1:03










  • $begingroup$
    To do that, don't we usually define $B = { x in A | x notin f(x) } $ ?
    $endgroup$
    – bluemuse
    Jan 21 at 1:09










  • $begingroup$
    Indeed, so $B$ is an element of $P(A)$ which is not in the range of $f$. In your question you alleged that we find some element of $A$ with some funny property.
    $endgroup$
    – bof
    Jan 21 at 1:16












  • $begingroup$
    Are you confusing elements with subsets? $B$ is a subset of $A$, which makes it an element of $P(A)$.
    $endgroup$
    – bof
    Jan 21 at 1:21










  • $begingroup$
    But what if we choose an f such that B is empty? I mean an f such that elements of A always map into subsets containing them so the property of B ( $ x notin f(x) $ ) is not possible? Yes I might be confusing a lot of stuff here because everyone seems to understand this besides me
    $endgroup$
    – bluemuse
    Jan 21 at 1:24














0












0








0





$begingroup$


I can intuitevely picture the power set of A to be of a greater cardinality than A, as it permits multiple combinations of elements of A. However, I couldn't understand the usual proof that comes with the theorem. I tried looking for other proofs but couldn't find any. The proof assumes that in any bijective function f we choose, there's going to be elements of A that map into subsets they're not member of in P(A). And then we would use these as a contradiction.



My problem is the following: what if we choose a function f such that elements of A only map into subsets they're member of? Is this possible? Am I missing something? Wouldn't the proof fail with this kind of, but my understanding is that this f shouldn't exist, then why?



Thank you!










share|cite|improve this question









$endgroup$




I can intuitevely picture the power set of A to be of a greater cardinality than A, as it permits multiple combinations of elements of A. However, I couldn't understand the usual proof that comes with the theorem. I tried looking for other proofs but couldn't find any. The proof assumes that in any bijective function f we choose, there's going to be elements of A that map into subsets they're not member of in P(A). And then we would use these as a contradiction.



My problem is the following: what if we choose a function f such that elements of A only map into subsets they're member of? Is this possible? Am I missing something? Wouldn't the proof fail with this kind of, but my understanding is that this f shouldn't exist, then why?



Thank you!







discrete-mathematics elementary-set-theory






share|cite|improve this question













share|cite|improve this question











share|cite|improve this question




share|cite|improve this question










asked Jan 21 at 0:51









bluemusebluemuse

976




976












  • $begingroup$
    I've never seen a proof like the one you describe. (Where did you find it?) The usual proof shows that, given any function $f:Ato P(A)$, we can find an element of $P(A)$ which is not in the range of $f$.
    $endgroup$
    – bof
    Jan 21 at 1:03










  • $begingroup$
    To do that, don't we usually define $B = { x in A | x notin f(x) } $ ?
    $endgroup$
    – bluemuse
    Jan 21 at 1:09










  • $begingroup$
    Indeed, so $B$ is an element of $P(A)$ which is not in the range of $f$. In your question you alleged that we find some element of $A$ with some funny property.
    $endgroup$
    – bof
    Jan 21 at 1:16












  • $begingroup$
    Are you confusing elements with subsets? $B$ is a subset of $A$, which makes it an element of $P(A)$.
    $endgroup$
    – bof
    Jan 21 at 1:21










  • $begingroup$
    But what if we choose an f such that B is empty? I mean an f such that elements of A always map into subsets containing them so the property of B ( $ x notin f(x) $ ) is not possible? Yes I might be confusing a lot of stuff here because everyone seems to understand this besides me
    $endgroup$
    – bluemuse
    Jan 21 at 1:24


















  • $begingroup$
    I've never seen a proof like the one you describe. (Where did you find it?) The usual proof shows that, given any function $f:Ato P(A)$, we can find an element of $P(A)$ which is not in the range of $f$.
    $endgroup$
    – bof
    Jan 21 at 1:03










  • $begingroup$
    To do that, don't we usually define $B = { x in A | x notin f(x) } $ ?
    $endgroup$
    – bluemuse
    Jan 21 at 1:09










  • $begingroup$
    Indeed, so $B$ is an element of $P(A)$ which is not in the range of $f$. In your question you alleged that we find some element of $A$ with some funny property.
    $endgroup$
    – bof
    Jan 21 at 1:16












  • $begingroup$
    Are you confusing elements with subsets? $B$ is a subset of $A$, which makes it an element of $P(A)$.
    $endgroup$
    – bof
    Jan 21 at 1:21










  • $begingroup$
    But what if we choose an f such that B is empty? I mean an f such that elements of A always map into subsets containing them so the property of B ( $ x notin f(x) $ ) is not possible? Yes I might be confusing a lot of stuff here because everyone seems to understand this besides me
    $endgroup$
    – bluemuse
    Jan 21 at 1:24
















$begingroup$
I've never seen a proof like the one you describe. (Where did you find it?) The usual proof shows that, given any function $f:Ato P(A)$, we can find an element of $P(A)$ which is not in the range of $f$.
$endgroup$
– bof
Jan 21 at 1:03




$begingroup$
I've never seen a proof like the one you describe. (Where did you find it?) The usual proof shows that, given any function $f:Ato P(A)$, we can find an element of $P(A)$ which is not in the range of $f$.
$endgroup$
– bof
Jan 21 at 1:03












$begingroup$
To do that, don't we usually define $B = { x in A | x notin f(x) } $ ?
$endgroup$
– bluemuse
Jan 21 at 1:09




$begingroup$
To do that, don't we usually define $B = { x in A | x notin f(x) } $ ?
$endgroup$
– bluemuse
Jan 21 at 1:09












$begingroup$
Indeed, so $B$ is an element of $P(A)$ which is not in the range of $f$. In your question you alleged that we find some element of $A$ with some funny property.
$endgroup$
– bof
Jan 21 at 1:16






$begingroup$
Indeed, so $B$ is an element of $P(A)$ which is not in the range of $f$. In your question you alleged that we find some element of $A$ with some funny property.
$endgroup$
– bof
Jan 21 at 1:16














$begingroup$
Are you confusing elements with subsets? $B$ is a subset of $A$, which makes it an element of $P(A)$.
$endgroup$
– bof
Jan 21 at 1:21




$begingroup$
Are you confusing elements with subsets? $B$ is a subset of $A$, which makes it an element of $P(A)$.
$endgroup$
– bof
Jan 21 at 1:21












$begingroup$
But what if we choose an f such that B is empty? I mean an f such that elements of A always map into subsets containing them so the property of B ( $ x notin f(x) $ ) is not possible? Yes I might be confusing a lot of stuff here because everyone seems to understand this besides me
$endgroup$
– bluemuse
Jan 21 at 1:24




$begingroup$
But what if we choose an f such that B is empty? I mean an f such that elements of A always map into subsets containing them so the property of B ( $ x notin f(x) $ ) is not possible? Yes I might be confusing a lot of stuff here because everyone seems to understand this besides me
$endgroup$
– bluemuse
Jan 21 at 1:24










3 Answers
3






active

oldest

votes


















1












$begingroup$

That's not what Cantor's diagonal argument says. What it says? We take a function from $A$ to $P(A)$. Then, for each element $x$ of $A$, we ask a question - is $x$ an element of $f(x)$? Construct a new set $Bsubset A$ as follows; its elements are precisely those $x$ for which we answered no. In other words, $xin B$ if $xnotin f(x)$ and $xnotin B$ if $xin f(x)$. This $B$ is a subset of $A$, so it's in $P(A)$. We then argue that it can't be in the image of $f$. If $yin B$, $ynotin f(y)$ by the definition of $B$, and $Bneq f(y)$. If $ynotin B$, $yin f(y)$ by the definition of $B$, and $Bneq f(y)$. Repeat over all $y$, and $B$ isn't in the image of $f$.



So, what if we choose $f$ such that $xin f(x)$ for all $xin A$? Then $B$ is empty, and it's not in the image of $f$ - because everything in the image of $f$ has at least one element.






share|cite|improve this answer









$endgroup$













  • $begingroup$
    This makes things a lot clearer! thank you!
    $endgroup$
    – bluemuse
    Jan 21 at 1:26



















0












$begingroup$

The proof I know goes as follows; Assume we have a Set A and a bijection f from A onto P(A). Now given this (fixed) bijection we may determine the subset of Elements of A whose projection( some subset of A) does not include themselves. Now there must be some element of A which is projected onto that subset since f is bijective. It can now be shown that this element is both a member and not a member of said subset, which is a contradiction.






share|cite|improve this answer









$endgroup$













  • $begingroup$
    Right, but $f$ doesn't have to be a bijection, just a surjection. The proof shows that there is no surjection from $A$ to $P(A)$.
    $endgroup$
    – bof
    Jan 21 at 1:19










  • $begingroup$
    " we may determine the subset of Elements of A whose projection( some subset of A) does not include themselves " this is exactly my question, how do we know such subset exists?
    $endgroup$
    – bluemuse
    Jan 21 at 1:21






  • 1




    $begingroup$
    @bluemuse Take for example the set of all integers which contain a seven. The set exists since for any integer we can clearly decide if it's a member of the set or not. The same principle is applied here.
    $endgroup$
    – Jagol95
    Jan 21 at 1:29






  • 1




    $begingroup$
    @bluemuse we are not assuming that this set is nonempty
    $endgroup$
    – Jagol95
    Jan 21 at 1:35



















0












$begingroup$

No, nothing goes wrong if we use such a map as our listing.



For example, one such map - arguably the simplest - is $$f:Arightarrow P(A):amapsto{a}.$$ The "antidiagonal set" associated to $f$ is $$Cantor(f)={a:anotin f(a)}={a:anotin{a}}=emptyset.$$ But this is fine: $Cantor(f)$ is indeed a subset of $A$, and it is indeed not in the range of $f$. So nothing's gone wrong.






share|cite|improve this answer









$endgroup$













    Your Answer





    StackExchange.ifUsing("editor", function () {
    return StackExchange.using("mathjaxEditing", function () {
    StackExchange.MarkdownEditor.creationCallbacks.add(function (editor, postfix) {
    StackExchange.mathjaxEditing.prepareWmdForMathJax(editor, postfix, [["$", "$"], ["\\(","\\)"]]);
    });
    });
    }, "mathjax-editing");

    StackExchange.ready(function() {
    var channelOptions = {
    tags: "".split(" "),
    id: "69"
    };
    initTagRenderer("".split(" "), "".split(" "), channelOptions);

    StackExchange.using("externalEditor", function() {
    // Have to fire editor after snippets, if snippets enabled
    if (StackExchange.settings.snippets.snippetsEnabled) {
    StackExchange.using("snippets", function() {
    createEditor();
    });
    }
    else {
    createEditor();
    }
    });

    function createEditor() {
    StackExchange.prepareEditor({
    heartbeatType: 'answer',
    autoActivateHeartbeat: false,
    convertImagesToLinks: true,
    noModals: true,
    showLowRepImageUploadWarning: true,
    reputationToPostImages: 10,
    bindNavPrevention: true,
    postfix: "",
    imageUploader: {
    brandingHtml: "Powered by u003ca class="icon-imgur-white" href="https://imgur.com/"u003eu003c/au003e",
    contentPolicyHtml: "User contributions licensed under u003ca href="https://creativecommons.org/licenses/by-sa/3.0/"u003ecc by-sa 3.0 with attribution requiredu003c/au003e u003ca href="https://stackoverflow.com/legal/content-policy"u003e(content policy)u003c/au003e",
    allowUrls: true
    },
    noCode: true, onDemand: true,
    discardSelector: ".discard-answer"
    ,immediatelyShowMarkdownHelp:true
    });


    }
    });














    draft saved

    draft discarded


















    StackExchange.ready(
    function () {
    StackExchange.openid.initPostLogin('.new-post-login', 'https%3a%2f%2fmath.stackexchange.com%2fquestions%2f3081354%2fproblem-understanding-the-cantor-theorems-proof%23new-answer', 'question_page');
    }
    );

    Post as a guest















    Required, but never shown

























    3 Answers
    3






    active

    oldest

    votes








    3 Answers
    3






    active

    oldest

    votes









    active

    oldest

    votes






    active

    oldest

    votes









    1












    $begingroup$

    That's not what Cantor's diagonal argument says. What it says? We take a function from $A$ to $P(A)$. Then, for each element $x$ of $A$, we ask a question - is $x$ an element of $f(x)$? Construct a new set $Bsubset A$ as follows; its elements are precisely those $x$ for which we answered no. In other words, $xin B$ if $xnotin f(x)$ and $xnotin B$ if $xin f(x)$. This $B$ is a subset of $A$, so it's in $P(A)$. We then argue that it can't be in the image of $f$. If $yin B$, $ynotin f(y)$ by the definition of $B$, and $Bneq f(y)$. If $ynotin B$, $yin f(y)$ by the definition of $B$, and $Bneq f(y)$. Repeat over all $y$, and $B$ isn't in the image of $f$.



    So, what if we choose $f$ such that $xin f(x)$ for all $xin A$? Then $B$ is empty, and it's not in the image of $f$ - because everything in the image of $f$ has at least one element.






    share|cite|improve this answer









    $endgroup$













    • $begingroup$
      This makes things a lot clearer! thank you!
      $endgroup$
      – bluemuse
      Jan 21 at 1:26
















    1












    $begingroup$

    That's not what Cantor's diagonal argument says. What it says? We take a function from $A$ to $P(A)$. Then, for each element $x$ of $A$, we ask a question - is $x$ an element of $f(x)$? Construct a new set $Bsubset A$ as follows; its elements are precisely those $x$ for which we answered no. In other words, $xin B$ if $xnotin f(x)$ and $xnotin B$ if $xin f(x)$. This $B$ is a subset of $A$, so it's in $P(A)$. We then argue that it can't be in the image of $f$. If $yin B$, $ynotin f(y)$ by the definition of $B$, and $Bneq f(y)$. If $ynotin B$, $yin f(y)$ by the definition of $B$, and $Bneq f(y)$. Repeat over all $y$, and $B$ isn't in the image of $f$.



    So, what if we choose $f$ such that $xin f(x)$ for all $xin A$? Then $B$ is empty, and it's not in the image of $f$ - because everything in the image of $f$ has at least one element.






    share|cite|improve this answer









    $endgroup$













    • $begingroup$
      This makes things a lot clearer! thank you!
      $endgroup$
      – bluemuse
      Jan 21 at 1:26














    1












    1








    1





    $begingroup$

    That's not what Cantor's diagonal argument says. What it says? We take a function from $A$ to $P(A)$. Then, for each element $x$ of $A$, we ask a question - is $x$ an element of $f(x)$? Construct a new set $Bsubset A$ as follows; its elements are precisely those $x$ for which we answered no. In other words, $xin B$ if $xnotin f(x)$ and $xnotin B$ if $xin f(x)$. This $B$ is a subset of $A$, so it's in $P(A)$. We then argue that it can't be in the image of $f$. If $yin B$, $ynotin f(y)$ by the definition of $B$, and $Bneq f(y)$. If $ynotin B$, $yin f(y)$ by the definition of $B$, and $Bneq f(y)$. Repeat over all $y$, and $B$ isn't in the image of $f$.



    So, what if we choose $f$ such that $xin f(x)$ for all $xin A$? Then $B$ is empty, and it's not in the image of $f$ - because everything in the image of $f$ has at least one element.






    share|cite|improve this answer









    $endgroup$



    That's not what Cantor's diagonal argument says. What it says? We take a function from $A$ to $P(A)$. Then, for each element $x$ of $A$, we ask a question - is $x$ an element of $f(x)$? Construct a new set $Bsubset A$ as follows; its elements are precisely those $x$ for which we answered no. In other words, $xin B$ if $xnotin f(x)$ and $xnotin B$ if $xin f(x)$. This $B$ is a subset of $A$, so it's in $P(A)$. We then argue that it can't be in the image of $f$. If $yin B$, $ynotin f(y)$ by the definition of $B$, and $Bneq f(y)$. If $ynotin B$, $yin f(y)$ by the definition of $B$, and $Bneq f(y)$. Repeat over all $y$, and $B$ isn't in the image of $f$.



    So, what if we choose $f$ such that $xin f(x)$ for all $xin A$? Then $B$ is empty, and it's not in the image of $f$ - because everything in the image of $f$ has at least one element.







    share|cite|improve this answer












    share|cite|improve this answer



    share|cite|improve this answer










    answered Jan 21 at 1:10









    jmerryjmerry

    9,8481225




    9,8481225












    • $begingroup$
      This makes things a lot clearer! thank you!
      $endgroup$
      – bluemuse
      Jan 21 at 1:26


















    • $begingroup$
      This makes things a lot clearer! thank you!
      $endgroup$
      – bluemuse
      Jan 21 at 1:26
















    $begingroup$
    This makes things a lot clearer! thank you!
    $endgroup$
    – bluemuse
    Jan 21 at 1:26




    $begingroup$
    This makes things a lot clearer! thank you!
    $endgroup$
    – bluemuse
    Jan 21 at 1:26











    0












    $begingroup$

    The proof I know goes as follows; Assume we have a Set A and a bijection f from A onto P(A). Now given this (fixed) bijection we may determine the subset of Elements of A whose projection( some subset of A) does not include themselves. Now there must be some element of A which is projected onto that subset since f is bijective. It can now be shown that this element is both a member and not a member of said subset, which is a contradiction.






    share|cite|improve this answer









    $endgroup$













    • $begingroup$
      Right, but $f$ doesn't have to be a bijection, just a surjection. The proof shows that there is no surjection from $A$ to $P(A)$.
      $endgroup$
      – bof
      Jan 21 at 1:19










    • $begingroup$
      " we may determine the subset of Elements of A whose projection( some subset of A) does not include themselves " this is exactly my question, how do we know such subset exists?
      $endgroup$
      – bluemuse
      Jan 21 at 1:21






    • 1




      $begingroup$
      @bluemuse Take for example the set of all integers which contain a seven. The set exists since for any integer we can clearly decide if it's a member of the set or not. The same principle is applied here.
      $endgroup$
      – Jagol95
      Jan 21 at 1:29






    • 1




      $begingroup$
      @bluemuse we are not assuming that this set is nonempty
      $endgroup$
      – Jagol95
      Jan 21 at 1:35
















    0












    $begingroup$

    The proof I know goes as follows; Assume we have a Set A and a bijection f from A onto P(A). Now given this (fixed) bijection we may determine the subset of Elements of A whose projection( some subset of A) does not include themselves. Now there must be some element of A which is projected onto that subset since f is bijective. It can now be shown that this element is both a member and not a member of said subset, which is a contradiction.






    share|cite|improve this answer









    $endgroup$













    • $begingroup$
      Right, but $f$ doesn't have to be a bijection, just a surjection. The proof shows that there is no surjection from $A$ to $P(A)$.
      $endgroup$
      – bof
      Jan 21 at 1:19










    • $begingroup$
      " we may determine the subset of Elements of A whose projection( some subset of A) does not include themselves " this is exactly my question, how do we know such subset exists?
      $endgroup$
      – bluemuse
      Jan 21 at 1:21






    • 1




      $begingroup$
      @bluemuse Take for example the set of all integers which contain a seven. The set exists since for any integer we can clearly decide if it's a member of the set or not. The same principle is applied here.
      $endgroup$
      – Jagol95
      Jan 21 at 1:29






    • 1




      $begingroup$
      @bluemuse we are not assuming that this set is nonempty
      $endgroup$
      – Jagol95
      Jan 21 at 1:35














    0












    0








    0





    $begingroup$

    The proof I know goes as follows; Assume we have a Set A and a bijection f from A onto P(A). Now given this (fixed) bijection we may determine the subset of Elements of A whose projection( some subset of A) does not include themselves. Now there must be some element of A which is projected onto that subset since f is bijective. It can now be shown that this element is both a member and not a member of said subset, which is a contradiction.






    share|cite|improve this answer









    $endgroup$



    The proof I know goes as follows; Assume we have a Set A and a bijection f from A onto P(A). Now given this (fixed) bijection we may determine the subset of Elements of A whose projection( some subset of A) does not include themselves. Now there must be some element of A which is projected onto that subset since f is bijective. It can now be shown that this element is both a member and not a member of said subset, which is a contradiction.







    share|cite|improve this answer












    share|cite|improve this answer



    share|cite|improve this answer










    answered Jan 21 at 1:12









    Jagol95Jagol95

    1637




    1637












    • $begingroup$
      Right, but $f$ doesn't have to be a bijection, just a surjection. The proof shows that there is no surjection from $A$ to $P(A)$.
      $endgroup$
      – bof
      Jan 21 at 1:19










    • $begingroup$
      " we may determine the subset of Elements of A whose projection( some subset of A) does not include themselves " this is exactly my question, how do we know such subset exists?
      $endgroup$
      – bluemuse
      Jan 21 at 1:21






    • 1




      $begingroup$
      @bluemuse Take for example the set of all integers which contain a seven. The set exists since for any integer we can clearly decide if it's a member of the set or not. The same principle is applied here.
      $endgroup$
      – Jagol95
      Jan 21 at 1:29






    • 1




      $begingroup$
      @bluemuse we are not assuming that this set is nonempty
      $endgroup$
      – Jagol95
      Jan 21 at 1:35


















    • $begingroup$
      Right, but $f$ doesn't have to be a bijection, just a surjection. The proof shows that there is no surjection from $A$ to $P(A)$.
      $endgroup$
      – bof
      Jan 21 at 1:19










    • $begingroup$
      " we may determine the subset of Elements of A whose projection( some subset of A) does not include themselves " this is exactly my question, how do we know such subset exists?
      $endgroup$
      – bluemuse
      Jan 21 at 1:21






    • 1




      $begingroup$
      @bluemuse Take for example the set of all integers which contain a seven. The set exists since for any integer we can clearly decide if it's a member of the set or not. The same principle is applied here.
      $endgroup$
      – Jagol95
      Jan 21 at 1:29






    • 1




      $begingroup$
      @bluemuse we are not assuming that this set is nonempty
      $endgroup$
      – Jagol95
      Jan 21 at 1:35
















    $begingroup$
    Right, but $f$ doesn't have to be a bijection, just a surjection. The proof shows that there is no surjection from $A$ to $P(A)$.
    $endgroup$
    – bof
    Jan 21 at 1:19




    $begingroup$
    Right, but $f$ doesn't have to be a bijection, just a surjection. The proof shows that there is no surjection from $A$ to $P(A)$.
    $endgroup$
    – bof
    Jan 21 at 1:19












    $begingroup$
    " we may determine the subset of Elements of A whose projection( some subset of A) does not include themselves " this is exactly my question, how do we know such subset exists?
    $endgroup$
    – bluemuse
    Jan 21 at 1:21




    $begingroup$
    " we may determine the subset of Elements of A whose projection( some subset of A) does not include themselves " this is exactly my question, how do we know such subset exists?
    $endgroup$
    – bluemuse
    Jan 21 at 1:21




    1




    1




    $begingroup$
    @bluemuse Take for example the set of all integers which contain a seven. The set exists since for any integer we can clearly decide if it's a member of the set or not. The same principle is applied here.
    $endgroup$
    – Jagol95
    Jan 21 at 1:29




    $begingroup$
    @bluemuse Take for example the set of all integers which contain a seven. The set exists since for any integer we can clearly decide if it's a member of the set or not. The same principle is applied here.
    $endgroup$
    – Jagol95
    Jan 21 at 1:29




    1




    1




    $begingroup$
    @bluemuse we are not assuming that this set is nonempty
    $endgroup$
    – Jagol95
    Jan 21 at 1:35




    $begingroup$
    @bluemuse we are not assuming that this set is nonempty
    $endgroup$
    – Jagol95
    Jan 21 at 1:35











    0












    $begingroup$

    No, nothing goes wrong if we use such a map as our listing.



    For example, one such map - arguably the simplest - is $$f:Arightarrow P(A):amapsto{a}.$$ The "antidiagonal set" associated to $f$ is $$Cantor(f)={a:anotin f(a)}={a:anotin{a}}=emptyset.$$ But this is fine: $Cantor(f)$ is indeed a subset of $A$, and it is indeed not in the range of $f$. So nothing's gone wrong.






    share|cite|improve this answer









    $endgroup$


















      0












      $begingroup$

      No, nothing goes wrong if we use such a map as our listing.



      For example, one such map - arguably the simplest - is $$f:Arightarrow P(A):amapsto{a}.$$ The "antidiagonal set" associated to $f$ is $$Cantor(f)={a:anotin f(a)}={a:anotin{a}}=emptyset.$$ But this is fine: $Cantor(f)$ is indeed a subset of $A$, and it is indeed not in the range of $f$. So nothing's gone wrong.






      share|cite|improve this answer









      $endgroup$
















        0












        0








        0





        $begingroup$

        No, nothing goes wrong if we use such a map as our listing.



        For example, one such map - arguably the simplest - is $$f:Arightarrow P(A):amapsto{a}.$$ The "antidiagonal set" associated to $f$ is $$Cantor(f)={a:anotin f(a)}={a:anotin{a}}=emptyset.$$ But this is fine: $Cantor(f)$ is indeed a subset of $A$, and it is indeed not in the range of $f$. So nothing's gone wrong.






        share|cite|improve this answer









        $endgroup$



        No, nothing goes wrong if we use such a map as our listing.



        For example, one such map - arguably the simplest - is $$f:Arightarrow P(A):amapsto{a}.$$ The "antidiagonal set" associated to $f$ is $$Cantor(f)={a:anotin f(a)}={a:anotin{a}}=emptyset.$$ But this is fine: $Cantor(f)$ is indeed a subset of $A$, and it is indeed not in the range of $f$. So nothing's gone wrong.







        share|cite|improve this answer












        share|cite|improve this answer



        share|cite|improve this answer










        answered Jan 21 at 1:16









        Noah SchweberNoah Schweber

        125k10150287




        125k10150287






























            draft saved

            draft discarded




















































            Thanks for contributing an answer to Mathematics Stack Exchange!


            • Please be sure to answer the question. Provide details and share your research!

            But avoid



            • Asking for help, clarification, or responding to other answers.

            • Making statements based on opinion; back them up with references or personal experience.


            Use MathJax to format equations. MathJax reference.


            To learn more, see our tips on writing great answers.




            draft saved


            draft discarded














            StackExchange.ready(
            function () {
            StackExchange.openid.initPostLogin('.new-post-login', 'https%3a%2f%2fmath.stackexchange.com%2fquestions%2f3081354%2fproblem-understanding-the-cantor-theorems-proof%23new-answer', 'question_page');
            }
            );

            Post as a guest















            Required, but never shown





















































            Required, but never shown














            Required, but never shown












            Required, but never shown







            Required, but never shown

































            Required, but never shown














            Required, but never shown












            Required, but never shown







            Required, but never shown







            Popular posts from this blog

            Mario Kart Wii

            What does “Dominus providebit” mean?

            Antonio Litta Visconti Arese